Respuesta :

I would say A because every other question it shows the exact amount but since it says 20-29 you can’t really say the answer by looking at the graph unless you guess but that’s not what the question is saying it’s saying which question cannot be answered by looking at the graph and I would Say You cannot answer A by looking at the graph.


Sorry if I am wrong

Answer and Step-by-step explanation:

The answer would be A.

This is because we don't know how many students correctly answered 29 questions, but we do know how many students correctly answered 20-29 questions on the test.

As for the other answers, it asks about the full range and not a specific number.

#teamtrees #PAW (Plant And Water)